Fórum de Matemática
DÚVIDAS? Nós respondemos!

Um Fórum em Português dedicado à Matemática
Data/Hora: 28 mar 2024, 10:00

Os Horários são TMG [ DST ]




Fazer Nova Pergunta Responder a este Tópico  [ 4 mensagens ] 
Autor Mensagem
MensagemEnviado: 02 Oct 2017, 14:10 
Offline

Registado: 27 set 2017, 08:00
Mensagens: 21
Localização: Brasil
Agradeceu: 10 vezes
Foi agradecido: 2 vezes
A questão está na Figura I e meus cálculos e minha resposta estão na Figura II. Os cálculos e o resultado estão corretos?

Obs.: Não fiz diretamente a integral entre -1 e 1, e sim entre 0 e 1, para depois multiplicar o resultado por dois, já que no ponto (0,0) não existe reta tangente. Isso também está correto?
Obs.2: Preciso dessa resolução o mais rápido possível. Tenho prova daqui a 2 horas.


Anexos:
Comentário do Ficheiro: Figura II - Resolução
Screenshot 2017-10-02 10.08.26.png
Screenshot 2017-10-02 10.08.26.png [ 168.5 KiB | Visualizado 1817 vezes ]
Comentário do Ficheiro: Figura I - Questão
Screenshot 2017-10-02 10.07.38.png
Screenshot 2017-10-02 10.07.38.png [ 84.3 KiB | Visualizado 1817 vezes ]
Topo
 Perfil  
 
MensagemEnviado: 02 Oct 2017, 14:37 
Offline

Registado: 07 mai 2016, 18:24
Mensagens: 260
Localização: Coimbra
Agradeceu: 8 vezes
Foi agradecido: 64 vezes
O raciocínio está correto. Vamos usar a Maxima para verificar os cálculos.
Código:
Maxima 5.38.1 http://maxima.sourceforge.net
using Lisp GNU Common Lisp (GCL) GCL 2.6.12
Distributed under the GNU Public License. See the file COPYING.
Dedicated to the memory of William Schelter.
The function bug_report() provides bug reporting information.
(%i1) x: t^3$ y: t^2$

(%i3) x^2 - y^3;
(%o3)                                  0
(%i4) integrate(sqrt(diff(x, t)^2 + diff(y, t)^2), t, 0, 1);
                                    3/2
                                  13      8
(%o4)                             ----- - --
                                   27     27
(%i5) float(%);
(%o5)                          1.43970987337155

Por alguma razão, a sua integral está duas vezes menor. Agora, donde vem 1/2? Temos
\(\int_0^1 t \sqrt{9t^2 + 4} \, dt = \frac{1}{18} \int_0^1 (9t^2 + 4)^{1/2} \, d(9t^2 + 4) = \frac{1}{18} \left. \frac{(9t^2 + 4)^{3/2}}{3/2} \right|_0^1\)

Boa sorte!

_________________
Não sou português. Não sou simpático.


Topo
 Perfil  
 
MensagemEnviado: 02 Oct 2017, 14:40 
Offline

Registado: 07 mai 2016, 18:24
Mensagens: 260
Localização: Coimbra
Agradeceu: 8 vezes
Foi agradecido: 64 vezes
MikeAlexBillsZ Escreveu:
Obs.: Não fiz diretamente a integral entre -1 e 1, e sim entre 0 e 1, para depois multiplicar o resultado por dois, já que no ponto (0,0) não existe reta tangente.


Na verdade podia integrar entre -1 e 1 que a sua parametrização é diferenciavel. O facto de a velocidade evanescer não importa.

_________________
Não sou português. Não sou simpático.


Topo
 Perfil  
 
MensagemEnviado: 04 Oct 2017, 15:46 
Offline

Registado: 27 set 2017, 08:00
Mensagens: 21
Localização: Brasil
Agradeceu: 10 vezes
Foi agradecido: 2 vezes
Obrigado, Etanislau. Me ajudou muito e a minha dúvida está sanada. Aquele 1/2 foi um descuido, mesmo. Não deveria estar lá. Consegui fazer a prova de segunda-feira com segurança, inclusive.


Topo
 Perfil  
 
Mostrar mensagens anteriores:  Ordenar por  
Fazer Nova Pergunta Responder a este Tópico  [ 4 mensagens ] 

Os Horários são TMG [ DST ]


Quem está ligado:

Utilizadores a ver este Fórum: Nenhum utilizador registado e 49 visitantes


Criar perguntas: Proibído
Responder a perguntas: Proibído
Editar Mensagens: Proibído
Apagar Mensagens: Proibído
Enviar anexos: Proibído

Pesquisar por:
Ir para:  
cron